Download Free Chevron Attorneys Recruitment Past Questions Practice – Updated

Download Free Chevron Attorneys Recruitment Past Questions Practice – Updated

Can you ace these Chevron Attorneys Recruitment Past Questions? Start honing your skills with some past questions practice and get a preview of what you can expect on test day.

We pulled these Chevron Attorneys Recruitment questions from our study pack   Chevron Attorneys Recruitment test Past questions study pack Each sample question includes correct answers, so you can see how to crack it!

Note about the test format: On Chevron Attorneys Recruitment test, you'll face 3 test sections:Numerical reasoning,Verbal reasoning, and Abstract reasoning questions, Logical Reasoning

Sample Chevron Attorneys Recruitment Test Past Questions and answers

LSAT Analytical Reasoning Practice Test 1
Answer the following question based on the below passage:

GAME 1
A landscape architect is selecting trees for a new project. The architect must choose between 7 types of trees—a Maple, a River Birch, an Oak, a Sycamore, Chestnut, a Blue Spruce, and a Pine. There are two separate areas, named area 1 and area 2, in which the trees will be planted. The trees must be planted in accordance with the following constraints:
Each area contains three different types of trees, and there are only three types of trees in each area.The same type of tree cannot be planted more than once in either area.
If an area contains a river birch, then the area cannot contain either a blue spruce or a pine.
If an area has a sycamore tree, it must also contain either a river birch or an oak.
Any area that contains an oak must also contain a maple.
Which of the following is an acceptable selection of trees for both areas?
A. (Area 1: Oak, Sycamore, Maple) (Area 2: Blue Spruce, Pine, River Birch)
B. (Area 1: Maple, Sycamore, River Birch) (Area 2: Blue Spruce, Chestnut, Oak)
C. (Area 1: Maple, River Birch, Chestnut) (Area 2: Sycamore, Oak, Blue Spruce)
D. (Area 1: Sycamore, River Birch, Maple) (Area 2: Blue Spruce, Pine, Chestnut)
E. (Area 1: Chestnut, Maple, Oak) (Area 2: Sycamore, River Birch, Chestnut)

__________________________________________________________________________________________

LSAT Analytical Reasoning Practice Test 2
Answer the following question based on the below passage:

GAME 1
A landscape architect is selecting trees for a new project. The architect must choose between 7 types of trees—a Maple, a River Birch, an Oak, a Sycamore, Chestnut, a Blue Spruce, and a Pine. There are two separate areas, named area 1 and area 2, in which the trees will be planted. The trees must be planted in accordance with the following constraints:
Each area contains three different types of trees, and there are only three types of trees in each area.
The same type of tree cannot be planted more than once in either area.
If an area contains a river birch, then the area cannot contain either a blue spruce or a pine.
If an area has a sycamore tree, it must also contain either a river birch or an oak.
Any area that contains an oak must also contain a maple.
Which of the following CANNOT be true?

A. A pine is not selected.
B. A Sycamore and a Maple are planted in area 2.
C. A river birch is not selected.
D. A blue spruce is in area 1 and a Pine is in area 2.
E. An oak and a chestnut tree are both in area 1.

___________________________________________________________________________________________

LSAT Logical Reasoning Practice Test 2
Answer the following question based on the below passage:
Good physicians inform patients of the possible risks associated with a procedure before the patient undergoes a procedure. Jolene’s physician didn’t warn her of the risks of her procedure before operating, so her physician is unethical.

Which of the following is an assumption that would allow the conclusion to be properly drawn?
A. If a physician does not warn a patient of the possible risks before the procedure, the physician is unethical.
B. All ethical physicians are good physicians
C. Jolene’s physician normally warns patients about the risks of procedures before she operates on her patients
D. Some physicians who are good physicians fail to act ethically in every circumstance.
E. All ethical physicians warn patients of risks before operating.

___________________________________________________________________________________________

LSAT Logical Reasoning Practice Test 4
Answer the following question based on the below passage:
Good physicians inform patients of the possible risks associated with a procedure before the patient undergoes a procedure. Jolene’s physician didn’t warn her of the risks of her procedure before operating, so her physician is unethical.

Which of the following is an assumption that would allow the conclusion to be properly drawn?
A. If a physician does not warn a patient of the possible risks before the procedure, the physician is unethical.
B. All ethical physicians are good physicians
C. Jolene’s physician normally warns patients about the risks of procedures before she operates on her patients
D. Some physicians who are good physicians fail to act ethically in every circumstance.
E. All ethical physicians warn patients of risks before operating.

TestStreams.com


If you need support on your study materials or just to ask us a question, You can chat with a Teststreams support agent using the live chat link below, or send us a quick in-mail.

WhatsApp Image 2023 03 28 at 10.49.10


Download Free Chevron Attorneys Recruitment Past Questions Practice – Updated

Want more questions like this? Get the Complete Chevron Attorneys Recruitment Past Questions Practice,  with even more questions and answers
Need help with a Questions pack, or any other material? Chat with our support team here